2 2/3 Divided by 1/4
Find the quotient simplify if possible

Answers

Answer 1

Answer:

Step-by-step explanation:

make 2 2/3 into an improper fraction by multiplying 2 by 3 and adding to 2

8/3

8/3 divided by 1/4 is the same as 8/3 multiplied by 4/1 (just 4)

[tex]\frac{8}{3}*\frac{4}{1} =\frac{32}{3}[/tex]

32/3=10 2/3

Answer 2
Hey there!

The simplified version of this expression is 32/3.

Hope this helps !

Related Questions

1. Warp coils, like those found on my new spaceship, are manufactured in groups of 36 (as everyone
knows). If there is a 4% rate of defect in manufacturing,
a) find the mean number of defects per group.
a)

Answers

Answer:

1.44

Step-by-step explanation:

No. of warp coils manufactured in one group = 36

defect rate = 4%

That is 4 out of every 100 warp coil is defective

No. of defective warp coil in one group = 4% of warp coil in one group

                                                             = 4% of 36 =4/100 *36

                                                             = 1.44

Thus, there are 1.44 defective coil in each group.

Since, mean number is average number of defect.

So if each group has 1.44 defective coils then mean for all the group will remain same i.e 1.44

I need the right answer please

Answers

Answer:

a) i) x = 6, y = 9

ii) 20

iii) -5

b) -40

Step-by-step explanation:

3(6) - 2(9)

18 - 18 = 0

greatest

3(10) - 2(5)

30 - 10

20

least

3(5) - 2(10)

15 - 20

-5

5(a - 4b)

5[-7 - 4(¼)]

5(-7 - 1)

5(-8)

-40

In the figure angle a measures 41 degrees and angle d measures 32. What is the measurement of angle E?

Answers

Answer:

it is c

Step-by-step explanation:

What is the total area, in square units, of the
figure shown below?

a. 6
b. 12
c. 24
d. 30

Answers

Answer:

30 !

Both are right triangles so you can do 4*3= 12 and then 12/2= 6

then 6*8=48 and 48/2=24

24+6 =30!

hope this helps ! :)

The answer is 30



8x6=48 48divided by 2 equals 24
3x4=12 12divided by 2equals 6
6+24 equals 30

Find the volume of this cylinder. Give your answer to 1 decimal place when the radius is 11 cm and the length is 14 cm

Answers

Answer:

5324

Step-by-step explanation:

volume of cylinder=πr^2l=22/7×11^2×14=5324

how do i solve this?

Answers

Answer:

idk

Step-by-step explanation:

idk

Answer:

that is to hard

Step-by-step explanation:

Big ideas math 6th grade chapter 5 test

Answers

Answer:

I'm not sure if you need help but...

Step-by-step explanation:

6+6=12

Can someone help me with questions 2 and 3 pls?

Answers

1 cup = 1/2 pint, so where it says 4 cups we'll just read 2 pints.

1.  In one show we serve 30 × 2 pints of small and 42 × 6 pints of regular.

Total = 30 × 2 + 42 × 6 =  60 + 252 = 312 pints

Answer: 312 pints

2. 8 pints per gallon, 2 gallons per container makes 16 pints per container.  

312 pints / 16 pints per container = 19.5 containers

We round up since we need a whole number of containers.

Answer: 20 containers

We can't really see question 3.

Justin played a game with 24 floating plastic ducks each with a colored sticker on the bottom. There are an equal number of red blue and green stickers on the set of ducks. What is the probability that one duck chosen at random will have a red or blue sticker

Answers

Answer:

Because there are 24 ducks total and they are separated equally into three categories, we can figure out that there are 24 / 3 = 8 ducks of each color. This makes the probability of pulling a red/blue duck would be (8*2)/24 or 16/24. If we want this to be a percentage we can just solve 16/24*100 and it will give us 66.66...%

What is the volume of this rectangular prism?

Answers

Answer:

volume = 5/8 cm^3

Step-by-step explanation:

v=lwh

(5/2)(1/3)(3/4)=15/24=5/8

Therefore, volume = 5/8 cm^3

I hope this helped and have a good rest of your day!

a rectangular field has side lengths that measure 9/10 mile and 1/2 mile. What is the area of the field

Answers

Answer:

9/20, or 0.45 mile.

Step-by-step explanation:[tex]\\[/tex]

To find the area of a rectangle, multiply length times width. In this problem we are given both the length and width.

[tex]\frac{9}{10}[/tex] × [tex]\frac{1\\}{2}[/tex] is just 9/20, which is 0.45.

I need help plzzzz I’m nearly done

Answers

Answer:

≈ 70.4 cm

Step-by-step explanation:

The circumference (C) of a circle is calculated as

C = 2πr ( r is the radius ) , thus

C = 2 × 3.142 × 11.2 ≈ 70.4 ( to 1 dec. place )

70.4 is the circumference

PLZZZ HELP ME I NEED HELP AND I HAVE A TIMER AND I WILL GIVE YOU BRAINLEST
On a coordinate plane, 2 lines intersect around (3.8, negative 1.5).
Given the graph of a system of equations, which statements are true about the solution? Check all that apply.
The x-value is an integer.
The x-value is between 3 and 4.
The y-value is between –2 and –1.
The x-value is positive.
The x- and y-values have the same sign

Answers

Answer:

b,c,d

Step-by-step explanation:

I just did the assignment

Answer:

b, c. d

Step-by-step explanation:

1) Which of the equivalent expressions for P(x) reveals the price which gives a profit of
zero without changing the form of the expression?
P(x) = -3x? + 432x - 1680
b) P(x) = -3(x-4)(x - 140)
P(x) = -3(x - 72)2 + 13872
2) Find a price, which gives a profit of zero.

Answers

Answer:

1. (B)P(x) = -3(x-4)(x - 140)

2. $140

Step-by-step explanation:

(1) Given the profit function, P(x) where x is the price, the expression which reveals the price which gives a profit of  zero without changing the form of the expression is:

P(x) = -3(x-4)(x - 140)

This is as a result of the fact that equating it to zero immediately yields the zeros of the quadratic polynomial.

(2)When the profit, P(x)=0, then using the expression above we have:

[tex]P(x) = -3(x-4)(x - 140)\\Since -3\neq 0, then:\\x-4=0$ or x-140=0\\x=4 or x=140[/tex]

A price of $4 or $140 as the case may be gives a profit of zero.

Ginger is taking a cab in Washington D.C. The taxi fare includes a base charge of $3.00 as soon as you enter the cab, plus an additional $2.50 per mile traveled. If Ginger starts off with a $50 bill, takes a cab for 8 miles and leaves a $2 tip. How much money will she be left with?

Answers

Answer:

she will be left with 25 dollars

Step-by-step explanation:

2.50 x 8=20

20+3+2=25

Answer:

Answer is 25 dollars.

Step-by-step explanation:

3$- 50 dollars= 47.

2.50 x 8= 20 dollars.

20$ -47 = 27.

2$- = 25

What is the solution to this system of equations?

Negative 3 x + 5 y = negative 2. 3 x + 7 y = 26.
(4, 2)
(2, 3 and one-third)
no solution
infinitely many solutions

Answers

Answer:

4,2

Step-by-step explanation:

did the test

The solution of the system of equation is (4, 2) which is correct option(A).

What is equation?

Equation is the defined as mathematical statements that have a minimum of two terms containing variables or numbers is equal.

Given the two systems of equations,

-3x + 5y = -2

3x + 7y = 26

Adding the both equations

-3x + 5y + 3x + 7y = -2+26

12y = 24

y = 2

Substitute the value of y in any one of the equations,

3x + 7 y = 26

3x + 7(2) = 26

3x = 26-14

3x = 12

x = 4

Hence, the solution of the system of equation is (4, 2).

Learn more about equation here:

brainly.com/question/10413253

#SPJ2

I need help with this math problem please

Answers

Answer:

x=12

Step-by-step explanation:

90 = x + 42 + 3x

48 = x + 3x

48 = 4x

12 = x

Answer:

x = 12

Step-by-step explanation:

Since we are given a right angle, we can say that:

sum of all angles = 90 degrees

x + 42 + 3x = 90

4x = 48

x = 12

So, x is equal to 12 degrees!

Hope this helps)))  ^‿^

Find the 76th term of the arithmetic sequence 17, 19, 21, ...

Answers

Answer: 167

Step-by-step explanation: To find the 76th term of this arithmetic sequence, we will be using our explicit formula which is shown below.

[tex]^{a}n = ^{a}1 (n - 1)d[/tex]

Since we want to determine the 76th term, we are going

to substitute 76 in for n in our explicit formula.

Then, [tex]^{a}1[/tex] will be our first term in our sequence which is 17.

Lastly, d is our common difference or the difference between each of the terms in our arithmetic sequence which is 2.

So we have [tex]^{a} 76 = 17 + (76 - 1)(2)[/tex].

Now we have all the information we need. Just simplify from here.

Make sure to apply order of operations because this is where many

students make mistakes. Inside the parentheses first!

(76 - 1) is going to be 75.

So we have [tex]^{a} 76 = 17 + (75)(2)[/tex].

Then, we have to make sure we multiply before we add.

So (75)(2) is going to give us 150.

So we have [tex]^{a} 76 = 17 + 150[/tex].

Now just add to find that [tex]^{a} 76 = 167[/tex].

So the 76th term is 167.

Von mixes tomato sauce and milk to make soup. First he puts 150 milliliters of
tomato sauce in a pot. Next he adds the milk that is in the measuring cup. How
many milliliters of soup does he make?
A. 400 milliliters
B. 460 mill
C. 350 mill
D. 500 mill

Answers

Answer:

500 milliliters or ml

Step-by-step explanation:

First, we find the amount of milk is in the measuring cup which is 350 ml  and then add 350 ml of milk to the 150 ml of tomato sauce to get 500 ml.

Hope it helped :)

The amount of soup made by Von is equal to 500 milliliters. Hence, option D is correct.

What are arithmetic Operations?

The four fundamental operations of arithmetic are addition, subtraction, multiplication, and division of two or even more items.

Included in them is the study of integers, especially the order of operations, which is important for all other areas of mathematics, notably algebra, data management, and geometry.

As per the data given in the question,

Von mixes tomato sauce and milk to make soup.

Initially, the amount of tomato sauce added in the pot = 150 milliliters.

Then, milk is added that is in the measuring cup that is 350 milliliters.

So,

The total amount of soup made by him,

= 150 + 350

= 500 milliliters.

To know more about arithmetic operations:

https://brainly.com/question/25277954

#SPJ5

The graph shows the rate that Tom travels
on his bicycle, with the line traveling through
the point (5,45)
Use this graph to answer questions 1 and 2.
What is the constant of proportionality in the graph?​

Answers

Answer:

The constant of proportionality is 9.

Step-by-step explanation:

k=y/x     This is the formula.

k=45/5   Plug in the x and y coordinates of a point on the line (5,45).

k=9         Here's your answer!

Pls answer quickly, this is due today !!!!!!
Mrs. Rappa has decided to take her $2,000 and go check out the savings accounts at Circle Bank of PI Haven. She picks up a flyer in the lobby. There are a couple of savings accounts to choose from. Look at the copy of the information from the flyer below. Write a letter to Mrs. Rappa and offer your advice to her about her options. Be sure to tell Mrs. Rappa your reasoning (using sentences and showing calculations for each account) to decide which account is best. If you have additional advice for Mrs. Rappa about the number of years she is investing her funds - you may do so.



Savings Accounts Interest Rate Years in account

Sleepy Savings 5% 7

Centsational Savings 10% 3


Answers

Answer:

Step-by-step explanation:

Dear Mrs.Rappa,

I don't know much about this but i do know that saving accounts are risky and she should just set up a checking. Because it's proven that you'll take the risk of losing value in your money. And you take a risk, but to help you more i need to know about what the 5% 7 means. if i do know what it's means, but the 5% is good, it's compound interest which means you get more money on that interest It grows, but as far as the 7 goes, IDK. but the saving %"s are good like yay money, but she takes the risks. So why the risk is there is because of the stock market, and also because the bank takes other clients money for them to pay other client's. So a risk in 7yrs.  

                                                       Sincerely,

                                                           First name Last name Here

A baseball coach purchased a total of 28 helmets and caps. Each helmet cost $21.50, and each cap cost $8.25. If the coach spent a total of $363.50, how many helmets did the coach purchase?

Answers

Answer:

5 helmets

31 caps

8.25, 16.50 24.75 33 x 10 = 330

330/2 = 165 = 20 helmets

363.50 - 165 = 198.50

5 x 21.50 = 107.50 (+ 11 x 8.25 = 16.50+24.75 +24.75+24.75 = 49.50+41.25) =£91 + 107.50 =  198.50

20+11 = 31

31 caps

5 helmets

Step-by-step explanation:

There are 10 helmets did the coach purchase.

What is an expression?

Mathematical expression is defined as the collection of the numbers variables and functions by using operations like addition, subtraction, multiplication, and division.

We have to given that;

A baseball coach purchased a total of 28 helmets and caps.

And, Each helmet cost $21.50, and each cap cost $8.25. and the coach spent a total of $363.50.

Let number of helmets = x

And, Number of caps = y

Here, A baseball coach purchased a total of 28 helmets and caps.

Hence, We get;

⇒ x + y = 28  ..(i)

And, Each helmet cost $21.50, and each cap cost $8.25. and the coach spent a total of $363.50.

Hence, We get;

⇒ 21.50x + 8.25y = 363.50

⇒ 5 (4.30x + 1.65y) = 5 × 72.7

⇒ 4.30x + 1.65y = 72.7  .. (ii)

Multiply by 4.30 in (i) ad subtract from (ii);

⇒ 4.30x + 4.30y - 4.30x - 1.65y = 120.4 - 72.7

⇒ 2.65y = 47.7

⇒ y = 18

And, From (i);

⇒ x + y = 28

⇒ x + 18 = 28

⇒ x = 10

Hence, Number of helmets = 10

Learn more about the mathematical expression visit:

brainly.com/question/1859113

#SPJ5

12. For angle A it is known that sin(A)>0 and tan (A) <0. If A is drawn in standard position, in which quadrant
does its terminal ray lie?​

Answers

Answer:

Quadrant 2

Step-by-step explanation:

Sin = opp/hyp and opp is positive in quadrants 1 & 2.

Tan = opp/adj - we need one of these values to be positive and the other negative. adj is negative in quadrants 2 & 3.

Quadrant 2 is found in both of the scenarios that we need to occur.

Find the probability that a point chosen at random will lie in the shaded region.

I don't understand what to do here :(

Answers

Step-by-step explanation:

In the 1st question

Total boxes = 6

Shaded boxes = 2

Probability = 2/6 = 1/3

What is the value of x in the equation below?

40x + 120 = 720

21

18

15

12.5

Answers

Answer:

The answer would be 15.

Step-by-step explanation:

The first thing that you have to do is subtract 120 on both sides

40x + 120 = 720

        -120     -120

40x = 600

Then you have to divide by 40.

40x= 600

600 ÷ 40 = 15

x = 15.

I hope I helped

Answer:

15

Step-by-step explanation:

40×15=600+120=720

find the term independent of x in the expansion of x^9(2-1/x^3)^6

(it's also pictured)

Answers

Answer:

20

Step-by-step explanation:

Write an expression in simplest form for the perimeter of a right triangle with leg lengths of 12a to the power of 5 and 9a to the power of 5.

Answers

Answer:

[tex]36a^5[/tex].

Step-by-step explanation:

In a right angle triangle,

Length of one leg [tex]=12a^5[/tex]

Length of other leg [tex]=9a^5[/tex]

According to the Pythagoras theorem,

[tex]Hypotenuse^2=leg_1^2+leg_2^2[/tex]

Using Pythagoras theorem,

[tex]Hypotenuse^2=(12a^5)^2+(9a^5)^2[/tex]

[tex]Hypotenuse^2=144(a^5)^2+81(a^5)^2[/tex]

[tex]Hypotenuse=\sqrt{225(a^5)^2}[/tex]

[tex]Hypotenuse=15a^5[/tex]

Perimeter of right angle triangle is the sum of all sides of the triangle.

[tex]Perimeter=12a^5+9a^5+15a^5[/tex]

[tex]Perimeter=36a^5[/tex]  units.

Therefore, the required expression is [tex]36a^5[/tex].

-4b - 5 + 2b =10. Can I have some help solving this.

Answers

Answer:

-7.5

Step-by-step explanation:

Simplify

-4b - 5 + 2b = 10

-2b - 5 = 10

-2b = 15

b = -7.5

Answer:

-7.5

Step-by-step explanation:

-4*- 7.5 =30

2*-7.5=   -15

30-5-15=10

HELP ASAP


table
———————————————
Radius (ft) Bushels of corn
500 2705
1000 10818
1500 24341
2000 43273
2500 67613


What type of function best models the given data set?

A. Constant
B. Exponential
C. Linear
D. Quadratic

Answers

Answer:

Linear would be fine as it doesn't decrease to under zero

and a curve graph would represent this data fine at 10818 turning point.

Step-by-step explanation:

f(n) = 41 – 5n
Complete the recursive formula of f(n).
f(1) =
f(n) = f(n − 1)+

Answers

Answer:

f(1) = 36

f(n) = f(n-1) - 5.

Step-by-step explanation:

f(1) = 41 - 5 = 36.

f(n) = f(n - 1) - 5.

Other Questions
A chessboard has 64 squares. George places 1 grain of rice on the first square, 2 grains on the second square, 4 grains on the third square, 8 grains on the fourth square, and so on, until he has placed grains of rice on 10 squares.Once George has put rice on the 10th square, he has placed a total of _____ grains of rice on the chess board. One brand of coffee is packaged in cylinders where the height is equal to the radius, r. The volume of the package, in cubic centimeters, can be found using the function V(r) = r3. The number of ounces of coffee in the cylinder depends on the volume of the cylinder, V, in cubic centimeters. This can be modeled by the function C(V) = 3.2V. Which function can be used to find the number of ounces of coffee in the can based on its radius? C(V(r)) = 32.768r3 C(V(r)) = 3.2r3 can someone help me please? A person exhibiting a recessive characteristic must be heterozygous for that gene. True or False How do the cells and organs of the Skeletal and Muscular system work together? 2/3 divided by 1 1/2 To whom does the car in the diagram most likely belong? East German citizen West German citizen Communist guard American guard Which word best describes the tone of this passage? A - critical B - apologetic C - objective D - discouraged During the first few years of a tree's life, its growth rate is linear. For example, an oak tree grows at a rate of 1.5 feet per year for the first twelve years. Mostafa plants a maple tree in his backyard and measures its height at 4 feet. Two years later, Mostafa measures the tree again and finds that its height is now 6.5 feet. Which statement is correct given this information? Running Co. had an equity investment where it owned less than 20% of an investee, and therefore Running Co. was not able to exercise significant influence. Information about the investment is below: 20X1 20X2 Investment cost 170,000 170,000 Fair value 181,400 155,000 Total unrealized gain (loss) 11,400 (15,000) The company sold the investment during 20X3 for the below price: Sales price 192,400 What is the gain (loss) recorded in the income statement in the year of sale, in 20X3 2 PointsWhich option best describes the role of citizens in a democracy? Question 2 options: The Galpagos tortoises are native to seven of the Galpagos Islands, a volcanic archipelago about 620 miles west of the Ecuadorian mainland. With lifespans in the wild of over ____________years, it is one of the longest-lived vertebrates. A captive individual lived at least ______________years! PLEASE HELP AGAIN, SORRY I SUCK AT MATH. what would happen to life if there were no decomposers Anyone got any tips I want a guniea pig really bhad! My parents are really annoying and telling me I got to wait till I move out. Please give me tips or ideas on how to convince them. Thank youBy the way thats me at my friends with her guinea pig After a nonstop day-long attack, how many men had Travis lost? How many strands of hair did the animators give elsa? math question down below screen shot Write a short dialouge in french between a judge and a lawyer . Make sure it has 5 lines with more than one word each. It can have phrases that describe the crime , a verdict, etc. Classify each representation shown below as either linear or exponential.xy0517.5211.25y = 2x + 4One person sends an email to 2 people. Those 2 people send it to 4 people, and those 4 send it to 8 people.